Finding all values of $n$ for which $1/n$ has a specific decimal period in different bases











up vote
0
down vote

favorite












I wanted to know if there is a way to easily determine the lowest value Natural number $n$ to where $1/n$ decimal actual period is purely repeating to $5$ digits in base $5$ and purely repeating to $4$ digits in base $6$?



What I learned so far



I had seen where $n$’s decimal representation independent of the base if the fraction is in lowest terms (is why I choose $1/n$) the period is limited to some $n-1$ or an integer $n-1$ is divisible by. It does not give the way to actually determine the period and it states the period may well be different for other bases.



For example $1/33$ could have periods of ($1,2,4,8,16,$ or $32$).



So my guessing point is lowest $n$ would need to be some multiple of $20$ then adding one also not having $2,3$ or $5$ as a prime factor to insure purely repeating for all chosen bases.



However I also saw for the denominator of $33$ that a period length of $10$ was given even though $33-1$ has no factors of $5$ in it so am left wondering if that theorem is actually true.










share|cite|improve this question




























    up vote
    0
    down vote

    favorite












    I wanted to know if there is a way to easily determine the lowest value Natural number $n$ to where $1/n$ decimal actual period is purely repeating to $5$ digits in base $5$ and purely repeating to $4$ digits in base $6$?



    What I learned so far



    I had seen where $n$’s decimal representation independent of the base if the fraction is in lowest terms (is why I choose $1/n$) the period is limited to some $n-1$ or an integer $n-1$ is divisible by. It does not give the way to actually determine the period and it states the period may well be different for other bases.



    For example $1/33$ could have periods of ($1,2,4,8,16,$ or $32$).



    So my guessing point is lowest $n$ would need to be some multiple of $20$ then adding one also not having $2,3$ or $5$ as a prime factor to insure purely repeating for all chosen bases.



    However I also saw for the denominator of $33$ that a period length of $10$ was given even though $33-1$ has no factors of $5$ in it so am left wondering if that theorem is actually true.










    share|cite|improve this question


























      up vote
      0
      down vote

      favorite









      up vote
      0
      down vote

      favorite











      I wanted to know if there is a way to easily determine the lowest value Natural number $n$ to where $1/n$ decimal actual period is purely repeating to $5$ digits in base $5$ and purely repeating to $4$ digits in base $6$?



      What I learned so far



      I had seen where $n$’s decimal representation independent of the base if the fraction is in lowest terms (is why I choose $1/n$) the period is limited to some $n-1$ or an integer $n-1$ is divisible by. It does not give the way to actually determine the period and it states the period may well be different for other bases.



      For example $1/33$ could have periods of ($1,2,4,8,16,$ or $32$).



      So my guessing point is lowest $n$ would need to be some multiple of $20$ then adding one also not having $2,3$ or $5$ as a prime factor to insure purely repeating for all chosen bases.



      However I also saw for the denominator of $33$ that a period length of $10$ was given even though $33-1$ has no factors of $5$ in it so am left wondering if that theorem is actually true.










      share|cite|improve this question















      I wanted to know if there is a way to easily determine the lowest value Natural number $n$ to where $1/n$ decimal actual period is purely repeating to $5$ digits in base $5$ and purely repeating to $4$ digits in base $6$?



      What I learned so far



      I had seen where $n$’s decimal representation independent of the base if the fraction is in lowest terms (is why I choose $1/n$) the period is limited to some $n-1$ or an integer $n-1$ is divisible by. It does not give the way to actually determine the period and it states the period may well be different for other bases.



      For example $1/33$ could have periods of ($1,2,4,8,16,$ or $32$).



      So my guessing point is lowest $n$ would need to be some multiple of $20$ then adding one also not having $2,3$ or $5$ as a prime factor to insure purely repeating for all chosen bases.



      However I also saw for the denominator of $33$ that a period length of $10$ was given even though $33-1$ has no factors of $5$ in it so am left wondering if that theorem is actually true.







      number-theory






      share|cite|improve this question















      share|cite|improve this question













      share|cite|improve this question




      share|cite|improve this question








      edited Nov 19 at 3:20









      Tianlalu

      2,9901936




      2,9901936










      asked Nov 9 at 23:49









      Toni Stack

      11




      11






















          1 Answer
          1






          active

          oldest

          votes

















          up vote
          0
          down vote













          For your first problem, the answer is that no such $n$ exists. Indeed, the base $6$ expansion having period $4$ tells you $n$ is a factor of $6^4-1=5times 7times 37$, and the base $5$ having period $5$ gives $n$ divides $5^5-1=2^2times 11times 71$. You can see these are incompatible constraints.



          Now your confusion with $n=33$ seems to come from applying a theorem outside its domain




          Theorem 1 Let $p$ be prime. The fundamental period of $1/p$ in base $b$ expansion, $b$ not divisible by $p$, is a factor of $p-1$.




          This need not work for $n$ composite. Indeed, the above is derived from




          Theorem 2 The minimal period of $1/n$ in base $b$ expansion, $gcd(n,b)=1$, is the multiplicative order of $b$ modulo $n$.




          Since the multiplicative group $(mathbb{Z}/p)^times$ has order $phi(p)=p-1$, this gives the case $n=p$ is prime.






          share|cite|improve this answer





















          • What is then the period constraints for an unreducable fraction (non factors of 3,11)/33 The table I thought was referring to fractions already in lowest terms
            – Toni Stack
            Nov 10 at 1:35






          • 1




            In general, the order has to be a divisor of $phi(n)$, where $phi$ is the Euler's totient function. But it is possible to give a slightly stronger bound -- it has to be a divisor of $operatorname{lcm}(phi(p_1^{r_1}),phi(p_2^{r_2}),dots,phi(p_k^{r_k}))$, where $n=p_1^{r_1}p_2^{r_2}dots p_k^{r_k}$. In your case, 33=3*11 and $phi(3)=2$, $phi(11)=10$ gives lcm =10.
            – user10354138
            Nov 10 at 2:44











          Your Answer





          StackExchange.ifUsing("editor", function () {
          return StackExchange.using("mathjaxEditing", function () {
          StackExchange.MarkdownEditor.creationCallbacks.add(function (editor, postfix) {
          StackExchange.mathjaxEditing.prepareWmdForMathJax(editor, postfix, [["$", "$"], ["\\(","\\)"]]);
          });
          });
          }, "mathjax-editing");

          StackExchange.ready(function() {
          var channelOptions = {
          tags: "".split(" "),
          id: "69"
          };
          initTagRenderer("".split(" "), "".split(" "), channelOptions);

          StackExchange.using("externalEditor", function() {
          // Have to fire editor after snippets, if snippets enabled
          if (StackExchange.settings.snippets.snippetsEnabled) {
          StackExchange.using("snippets", function() {
          createEditor();
          });
          }
          else {
          createEditor();
          }
          });

          function createEditor() {
          StackExchange.prepareEditor({
          heartbeatType: 'answer',
          convertImagesToLinks: true,
          noModals: true,
          showLowRepImageUploadWarning: true,
          reputationToPostImages: 10,
          bindNavPrevention: true,
          postfix: "",
          imageUploader: {
          brandingHtml: "Powered by u003ca class="icon-imgur-white" href="https://imgur.com/"u003eu003c/au003e",
          contentPolicyHtml: "User contributions licensed under u003ca href="https://creativecommons.org/licenses/by-sa/3.0/"u003ecc by-sa 3.0 with attribution requiredu003c/au003e u003ca href="https://stackoverflow.com/legal/content-policy"u003e(content policy)u003c/au003e",
          allowUrls: true
          },
          noCode: true, onDemand: true,
          discardSelector: ".discard-answer"
          ,immediatelyShowMarkdownHelp:true
          });


          }
          });














          draft saved

          draft discarded


















          StackExchange.ready(
          function () {
          StackExchange.openid.initPostLogin('.new-post-login', 'https%3a%2f%2fmath.stackexchange.com%2fquestions%2f2992088%2ffinding-all-values-of-n-for-which-1-n-has-a-specific-decimal-period-in-diffe%23new-answer', 'question_page');
          }
          );

          Post as a guest















          Required, but never shown

























          1 Answer
          1






          active

          oldest

          votes








          1 Answer
          1






          active

          oldest

          votes









          active

          oldest

          votes






          active

          oldest

          votes








          up vote
          0
          down vote













          For your first problem, the answer is that no such $n$ exists. Indeed, the base $6$ expansion having period $4$ tells you $n$ is a factor of $6^4-1=5times 7times 37$, and the base $5$ having period $5$ gives $n$ divides $5^5-1=2^2times 11times 71$. You can see these are incompatible constraints.



          Now your confusion with $n=33$ seems to come from applying a theorem outside its domain




          Theorem 1 Let $p$ be prime. The fundamental period of $1/p$ in base $b$ expansion, $b$ not divisible by $p$, is a factor of $p-1$.




          This need not work for $n$ composite. Indeed, the above is derived from




          Theorem 2 The minimal period of $1/n$ in base $b$ expansion, $gcd(n,b)=1$, is the multiplicative order of $b$ modulo $n$.




          Since the multiplicative group $(mathbb{Z}/p)^times$ has order $phi(p)=p-1$, this gives the case $n=p$ is prime.






          share|cite|improve this answer





















          • What is then the period constraints for an unreducable fraction (non factors of 3,11)/33 The table I thought was referring to fractions already in lowest terms
            – Toni Stack
            Nov 10 at 1:35






          • 1




            In general, the order has to be a divisor of $phi(n)$, where $phi$ is the Euler's totient function. But it is possible to give a slightly stronger bound -- it has to be a divisor of $operatorname{lcm}(phi(p_1^{r_1}),phi(p_2^{r_2}),dots,phi(p_k^{r_k}))$, where $n=p_1^{r_1}p_2^{r_2}dots p_k^{r_k}$. In your case, 33=3*11 and $phi(3)=2$, $phi(11)=10$ gives lcm =10.
            – user10354138
            Nov 10 at 2:44















          up vote
          0
          down vote













          For your first problem, the answer is that no such $n$ exists. Indeed, the base $6$ expansion having period $4$ tells you $n$ is a factor of $6^4-1=5times 7times 37$, and the base $5$ having period $5$ gives $n$ divides $5^5-1=2^2times 11times 71$. You can see these are incompatible constraints.



          Now your confusion with $n=33$ seems to come from applying a theorem outside its domain




          Theorem 1 Let $p$ be prime. The fundamental period of $1/p$ in base $b$ expansion, $b$ not divisible by $p$, is a factor of $p-1$.




          This need not work for $n$ composite. Indeed, the above is derived from




          Theorem 2 The minimal period of $1/n$ in base $b$ expansion, $gcd(n,b)=1$, is the multiplicative order of $b$ modulo $n$.




          Since the multiplicative group $(mathbb{Z}/p)^times$ has order $phi(p)=p-1$, this gives the case $n=p$ is prime.






          share|cite|improve this answer





















          • What is then the period constraints for an unreducable fraction (non factors of 3,11)/33 The table I thought was referring to fractions already in lowest terms
            – Toni Stack
            Nov 10 at 1:35






          • 1




            In general, the order has to be a divisor of $phi(n)$, where $phi$ is the Euler's totient function. But it is possible to give a slightly stronger bound -- it has to be a divisor of $operatorname{lcm}(phi(p_1^{r_1}),phi(p_2^{r_2}),dots,phi(p_k^{r_k}))$, where $n=p_1^{r_1}p_2^{r_2}dots p_k^{r_k}$. In your case, 33=3*11 and $phi(3)=2$, $phi(11)=10$ gives lcm =10.
            – user10354138
            Nov 10 at 2:44













          up vote
          0
          down vote










          up vote
          0
          down vote









          For your first problem, the answer is that no such $n$ exists. Indeed, the base $6$ expansion having period $4$ tells you $n$ is a factor of $6^4-1=5times 7times 37$, and the base $5$ having period $5$ gives $n$ divides $5^5-1=2^2times 11times 71$. You can see these are incompatible constraints.



          Now your confusion with $n=33$ seems to come from applying a theorem outside its domain




          Theorem 1 Let $p$ be prime. The fundamental period of $1/p$ in base $b$ expansion, $b$ not divisible by $p$, is a factor of $p-1$.




          This need not work for $n$ composite. Indeed, the above is derived from




          Theorem 2 The minimal period of $1/n$ in base $b$ expansion, $gcd(n,b)=1$, is the multiplicative order of $b$ modulo $n$.




          Since the multiplicative group $(mathbb{Z}/p)^times$ has order $phi(p)=p-1$, this gives the case $n=p$ is prime.






          share|cite|improve this answer












          For your first problem, the answer is that no such $n$ exists. Indeed, the base $6$ expansion having period $4$ tells you $n$ is a factor of $6^4-1=5times 7times 37$, and the base $5$ having period $5$ gives $n$ divides $5^5-1=2^2times 11times 71$. You can see these are incompatible constraints.



          Now your confusion with $n=33$ seems to come from applying a theorem outside its domain




          Theorem 1 Let $p$ be prime. The fundamental period of $1/p$ in base $b$ expansion, $b$ not divisible by $p$, is a factor of $p-1$.




          This need not work for $n$ composite. Indeed, the above is derived from




          Theorem 2 The minimal period of $1/n$ in base $b$ expansion, $gcd(n,b)=1$, is the multiplicative order of $b$ modulo $n$.




          Since the multiplicative group $(mathbb{Z}/p)^times$ has order $phi(p)=p-1$, this gives the case $n=p$ is prime.







          share|cite|improve this answer












          share|cite|improve this answer



          share|cite|improve this answer










          answered Nov 10 at 0:30









          user10354138

          6,9251624




          6,9251624












          • What is then the period constraints for an unreducable fraction (non factors of 3,11)/33 The table I thought was referring to fractions already in lowest terms
            – Toni Stack
            Nov 10 at 1:35






          • 1




            In general, the order has to be a divisor of $phi(n)$, where $phi$ is the Euler's totient function. But it is possible to give a slightly stronger bound -- it has to be a divisor of $operatorname{lcm}(phi(p_1^{r_1}),phi(p_2^{r_2}),dots,phi(p_k^{r_k}))$, where $n=p_1^{r_1}p_2^{r_2}dots p_k^{r_k}$. In your case, 33=3*11 and $phi(3)=2$, $phi(11)=10$ gives lcm =10.
            – user10354138
            Nov 10 at 2:44


















          • What is then the period constraints for an unreducable fraction (non factors of 3,11)/33 The table I thought was referring to fractions already in lowest terms
            – Toni Stack
            Nov 10 at 1:35






          • 1




            In general, the order has to be a divisor of $phi(n)$, where $phi$ is the Euler's totient function. But it is possible to give a slightly stronger bound -- it has to be a divisor of $operatorname{lcm}(phi(p_1^{r_1}),phi(p_2^{r_2}),dots,phi(p_k^{r_k}))$, where $n=p_1^{r_1}p_2^{r_2}dots p_k^{r_k}$. In your case, 33=3*11 and $phi(3)=2$, $phi(11)=10$ gives lcm =10.
            – user10354138
            Nov 10 at 2:44
















          What is then the period constraints for an unreducable fraction (non factors of 3,11)/33 The table I thought was referring to fractions already in lowest terms
          – Toni Stack
          Nov 10 at 1:35




          What is then the period constraints for an unreducable fraction (non factors of 3,11)/33 The table I thought was referring to fractions already in lowest terms
          – Toni Stack
          Nov 10 at 1:35




          1




          1




          In general, the order has to be a divisor of $phi(n)$, where $phi$ is the Euler's totient function. But it is possible to give a slightly stronger bound -- it has to be a divisor of $operatorname{lcm}(phi(p_1^{r_1}),phi(p_2^{r_2}),dots,phi(p_k^{r_k}))$, where $n=p_1^{r_1}p_2^{r_2}dots p_k^{r_k}$. In your case, 33=3*11 and $phi(3)=2$, $phi(11)=10$ gives lcm =10.
          – user10354138
          Nov 10 at 2:44




          In general, the order has to be a divisor of $phi(n)$, where $phi$ is the Euler's totient function. But it is possible to give a slightly stronger bound -- it has to be a divisor of $operatorname{lcm}(phi(p_1^{r_1}),phi(p_2^{r_2}),dots,phi(p_k^{r_k}))$, where $n=p_1^{r_1}p_2^{r_2}dots p_k^{r_k}$. In your case, 33=3*11 and $phi(3)=2$, $phi(11)=10$ gives lcm =10.
          – user10354138
          Nov 10 at 2:44


















          draft saved

          draft discarded




















































          Thanks for contributing an answer to Mathematics Stack Exchange!


          • Please be sure to answer the question. Provide details and share your research!

          But avoid



          • Asking for help, clarification, or responding to other answers.

          • Making statements based on opinion; back them up with references or personal experience.


          Use MathJax to format equations. MathJax reference.


          To learn more, see our tips on writing great answers.





          Some of your past answers have not been well-received, and you're in danger of being blocked from answering.


          Please pay close attention to the following guidance:


          • Please be sure to answer the question. Provide details and share your research!

          But avoid



          • Asking for help, clarification, or responding to other answers.

          • Making statements based on opinion; back them up with references or personal experience.


          To learn more, see our tips on writing great answers.




          draft saved


          draft discarded














          StackExchange.ready(
          function () {
          StackExchange.openid.initPostLogin('.new-post-login', 'https%3a%2f%2fmath.stackexchange.com%2fquestions%2f2992088%2ffinding-all-values-of-n-for-which-1-n-has-a-specific-decimal-period-in-diffe%23new-answer', 'question_page');
          }
          );

          Post as a guest















          Required, but never shown





















































          Required, but never shown














          Required, but never shown












          Required, but never shown







          Required, but never shown

































          Required, but never shown














          Required, but never shown












          Required, but never shown







          Required, but never shown







          Popular posts from this blog

          Biblatex bibliography style without URLs when DOI exists (in Overleaf with Zotero bibliography)

          ComboBox Display Member on multiple fields

          Is it possible to collect Nectar points via Trainline?